Danessa needs to compare the area of one large circle with a diameter of 8 to the total area of 2 smalle

diameter one-half that of the large circle.

Which statements about the areas are true? Select three options.

The area of the large circle is 16

The area of one small circle is 4

The area of one small circle will be one-half of the area of the large circle.

The total area of the two small circles will equal that of the large circle

The total area of the two small circles will be one-half of the area of the large circle

Answers

Answer 1

Answer:

The area of the large circle is 16π

The area of one small circle is 4π

The total area of the two small circles will be one-half of the area of the large circle

Step by step explanation:

Area if circle = πr²

area of one large circle with a diameter of 8:

r = diameter/2 = 8/2 = 4

Area = π×4² = 16π

total area of 2 smaller diameter one-half that of the large circle.

Area = πr²

Diameter of small circle = 1/2(bigger circle diameter)

Diameter = 8/2 = 4

radius = 4/2 = 2

Area of one small circle = π×(2)² = 4π

Total Area of Two smaller circles= 2(4π) = 8π

Area of two smaller circle = 1/2(area of bigger circle) = 1/2(16π) 8π

Therefore, based on the answer:

The area of the large circle is 16π

The area of one small circle is 4π

The total area of the two small circles will be one-half of the area of the large circle

Answer 2

Answer:

The area of the large circle is 16 pi.

The area of one small circle is 4 pi.

The total area of the two small circles will be one-half of the area of the large circle.

Step-by-step explanation:

To shorten this down for ya lazy people like me the answers are 1, 2, and 5!!!

I got this right on my UNit test review!!!!!

I hope this helps!!!!!!


Related Questions

An employer uses the linear regression equation y = 0.18 x + 320.22 to predict the weekly salary, y, of an employee who sells x dollars worth of merchandise. Last week, Joaquin sold $1500 worth of merchandise. The same week, Alex earned $650. Using the regression equation, which is an accurate comparison? Alex sold merchandise worth about $60 more than what Joaquin sold. Joaquin sold merchandise worth about $60 more than what Joaquin sold. Alex earned about $60 more than Joaquin did. Joaquin earned about $60 more than Alex did.

Answers

Answer:

Option C is correct.

Alex earned about $60 more than Joaquin did.

Step-by-step explanation:

The linear regression equation

y = 0.18 x + 320.22

is used to predict the weekly salary, y, of an employee who sells x dollars worth of merchandise.

Joaquin sold $1500 worth of goods. Meaning that x for that week for Joaquin is 1500.

Joaquin' s salary for that week is then given as

y = 0.18x + 320.22

y = 0.18(1500) + 320.22 = 590.22

Hence, Joaquin's salary for that week = $590.22

Alex earns $650 that week. Meaning that y for Alex in that week = 650

y = 0.18x + 320.22

650 = 0.18x + 320.22

0.18x = 650 - 320.22 = 329.78

x = (329.78/0.18) = 1832.1

Hence, Alex sold goods worth $1832.1 that week.

Joaquin sold goods worth $1500

Joaquin earned $590.22

Alex sold goods worth $1832.1

Alex earned $650

From this calculation, it is evident that 'Alex earned about $60 more than Joaquin did' is the correct option as $650 is about $60 more than $590.22

Hope this Helps!!!

Answer:

the answer is C :)

Step-by-step explanation:

A paint tin is leaking, a circular
puddle is formed and the radius of the
circle increases at a constant rate.
a) If the circumference of the circle
is increasing at a rate of 12cm/s, find
the rate at which the radius is increasing.​

Answers

Answer:

The rate at which the radius is increasing

dr/dt = 1.91 cm/s

Step-by-step explanation:

The circumference C of a circle can be written as;

C = 2πr .....1

Where;

r = radius

The rate at which the circumference of the circle

is increasing can be written as dC/dt;

Differentiating equation 1, we have;

dC/dt = 2π dr/dt

Making dr/dt the subject of formula;

dr/dt = (dC/dt)/2π

Given;

dC/dt = 12cm/s

Substituting the value of dC/dt;

dr/dt = 12/2π

dr/dt = 1.909859317102 = 1.91 cm/s

The rate at which the radius is increasing dr/dt is 1.91 cm/s

Which equation will solve the following word problem? One small pitcher and one large pitcher can hold 12 cups of water. If the large pitcher is twice the size of the small pitcher, how much does the small pitcher hold? 2S + S = 12 2S = 12/S 2S - S = 12 S = 12/2S

Answers

Answer:

Option (1).

Step-by-step explanation:

Amount of water hold by both the pitchers (Large + small) = 12 cups

Let the small pitcher holds the water = S cups

If the water hold by the large pitcher is twice the size of the small pitcher, then the amount of water hold by large pitcher = 2S

Total amount water hold by both = S + 2S

Equation for this situation will be,

S + 2S = 12

Therefore, Option (1) will be the answer.

Please answer this multiple choice question !! Thank u !! Will give brainliest!!

Answers

Answer:

B: 3/2x - 7 + 3 = 0

Step-by-step explanation:

Answer: C

Step-by-step explanation:

General form is in Ax+By+C. To do this, you move everything onto one side.

[tex]\frac{3}{2}x -y+3=0[/tex]

This should be the general form, but this is not correct in these answer choices! Let's try this another way to get Ax+By+C by making Ax not a fraction.

[tex]y=\frac{3}{2} x-3[/tex]

[tex]y+3=\frac{3}{2} x[/tex]

[tex]2(y+3)=3x[/tex]

[tex]2y+6=3x[/tex]

[tex]3x-2y-6=0[/tex]

This may seem incorrect, but we can always check our answer by plugging in numbers.

In our slope intercept form, let's say x=2. What would be our y?

[tex]y=\frac{3}{2} (2)-3[/tex]

[tex]y=3-3[/tex]

[tex]y=0[/tex]

Now we know our coordinates should be (2, 0). When we rewrite the slope-intercept form, we should still get 0 when we plug in x=2 and y=0. Let's check on all answer choices to see which one works.

A. Correct

-3(2)+2(0)+6

-6-6=0

0=0

B. Incorrect

3/2(2)-(0)+3

3+3=6

6≠0

C. Correct

3(2)-2(0)-6

6-6=0

0=0

D. Incorrect; not general form

2(0)=3(2)-6

0=6-6

Now, you can see that A and C are both correct, but the main difference, is that you want to make A positive. The only option where A is positive is in answer C.

What’s the correct answer for this question?

Answers

Answer:

C:

Step-by-step explanation:

In the attached file

Which of the following equations is equivalent to S = pi r squared h?

Answers

Answer:

B. h=S/πr²

Step-by-step explanation:

The question lacks options. Here is the complete question.

Which of the following equations is equivalent to S = πr²h

a. h=S-πr^2

b. h=S/πR^2

C. h= πr^2/S

D. h=S+ πr^2

To know the equation equivalent to πr²h, we will make h the subject of the formula as shown from the one given in equation.

S = πr²h

To get h, we will divide both sides by the coefficient of h (i.e πr²)

S/πr² = πr²h/πr²

S/πr² = h

h = S/πr²

This shows that h = S/πr² is equivalent to S = πr²h

Potatoes cost Janice $1.10 per pound, and she has $6.00 that she could possibly spend on potatoes or other items. If she feels that the first pound of potatoes is worth $1.50, the second pound is worth $1.14, the third pound is worth $1.05, and all subsequent pounds are worth $0.30, how many pounds of potatoes will she purchase? What if she only had $3.00 to spend?

Answers

Answer:

she will purchase 8 pounds of potatoes.

But if she had on $3.0

She will purchase only two pounds

Step-by-step explanation:

first pound of potatoes is worth $1.50, the second pound is worth $1.14, the third pound is worth $1.05, and all subsequent pounds are worth $0.30.

1.5+1.14+1.05+0.30=$ 3.99

6-3.99= 2.01

If the rest cost 0.5

Then there are 4 pounds of 0.5 in 2.01.

So total she will purchase 8 pounds of potatoes.

But if she had on $3.0

She will purchase only two pounds, as it will only purchase1.5+1.14+= 2.64 worth of potatoes.

With $6 Janice can get a total of 10.7 pounds, but with $3  she will be able to afford the first two pounds only

Given Data

Amount at hand = $6

First Pound cost = $1.50

Second Pound cost = $1.14

Third Pound cost = $1.05

Total = 1.5+1.14+1.05 =  3.69

Balance  = 6-3.69 = $2.31

Hence the number of pound we can get with $2.31 for subsequent purchase will be

= 2.31/0.3

= 7.7 pounds

This means that the total number of pounds will be

= 3+7.7

=10.7pounds

subsequent Pound cost = $0.3

Learn more about algebra here:

https://brainly.com/question/6143254

surface area, will mark brainliest.

Answers

Answer: 30 in^2

Step-by-step explanation:

What’s the correct answer for this question?

Answers

Answer:

B

Step-by-step explanation:

I put the answer in an atachement

Answer:

B.

Step-by-step explanation:

In the attached file

PLEASE HELP NO ONE IS HELPING

Answers

Answer:

-7 goes in the box

Step-by-step explanation:

x^ -3 = x^4 * x^n

We know that a^b * a^c = a^ (b+c)

x^-3 = x^(4+n)

When the bases are the same the exponents are the same

-3 = 4+n

Subtract 4 from each side

-3 -4 = 4+n-4

-7 = n

Find the value when x= 2 and y =3.
2x⁰ y-²
1/9
2/9
-18​

Answers

Answer:

36

Step-by-step explanation:

Answer:

The answer is 2/9

Step-by-step explanation:

2(2)^0 (3)^-2 = 2(1)/(3)^2 = 2/9

What are the factors of x2 + 4x + 3?
A X3 - 1
B
4x2 + 3
C (x - 3)(x + 1)
D (x+3)(x + 1)

Answers

Answer:

D. (X + 3 ) ( X +1 )

process:

x² + 4x + 3

= x² + (3 + 1)x +3

= x² + 3x + x + 3

= X (x+3) + 1 (X+3)

= (X + 3 ) ( X +1 )

The answer is D (x+3) (x+1)

In a survey, 376 out of 1,078 US adults said they drink at least 4 cups of coffee a day. Find a point estimate (P) for the population proportion of US adults who drink at least 4 cups of coffee a day, then construct a 99% confidence interval for the proportion of adults who drink at least 4 cups of coffee a day.

Answers

Answer:

The point estimate is 0.3488.

The 99% confidence interval for the proportion of adults who drink at least 4 cups of coffee a day is (0.3114, 0.3862).

Step-by-step explanation:

In a sample with a number n of people surveyed with a point estimate of [tex]\pi[/tex], and a confidence level of [tex]1-\alpha[/tex], we have the following confidence interval of proportions.

[tex]\pi \pm z\sqrt{\frac{\pi(1-\pi)}{n}}[/tex]

In which

z is the zscore that has a pvalue of [tex]1 - \frac{\alpha}{2}[/tex].

For this problem, we have that:

[tex]n = 1078, \pi = \frac{376}{1078} = 0.3488[/tex]

The point estimate is 0.3488.

99% confidence level

So [tex]\alpha = 0.01[/tex], z is the value of Z that has a pvalue of [tex]1 - \frac{0.01}{2} = 0.995[/tex], so [tex]Z = 2.575[/tex].

The lower limit of this interval is:

[tex]\pi - z\sqrt{\frac{\pi(1-\pi)}{n}} = 0.3488 - 2.575\sqrt{\frac{0.3488*0.6512}{1078}} = 0.3114[/tex]

The upper limit of this interval is:

[tex]\pi + z\sqrt{\frac{\pi(1-\pi)}{n}} = 0.3488 + 2.575\sqrt{\frac{0.3488*0.6512}{1078}} = 0.3862[/tex]

The 99% confidence interval for the proportion of adults who drink at least 4 cups of coffee a day is (0.3114, 0.3862).

A sports company wants to package a ball with a 1.5-inch radius in sets of two. They have two options: a cylinder or a square prism. 2 balls are inside of a cylinder and 2 balls are inside of a square prism. The cylinder has a height of 6 inches and a radius of 1.5 inches. The square prism has base lengths of 3 inches and the prism has a height of 6 inches. The company wants to use the package that has the least amount of wasted space. The company should choose

Answers

Answer:

C. the cylinder because it has approximately 11.6 in.3 less wasted space than the prism.

The company should choose the cylinder because it has approximately 11.6 in.3 less wasted space than the prism.

We have given that,

A sports company wants to package a ball with a 1.5-inch radius in sets of two.

They have two options a cylinder or a square prism.

2 balls are inside of a cylinder and 2 balls are inside of a square prism. The cylinder has a height of 6 inches and a radius of 1.5 inches.

What is the square prism?

A square prism is basically a cuboid, that has square bases. It has four rectangular faces and two square-shaped ends. In Geometry, we have studied various three-dimensional shapes called solid shapes or solids.

The square prism has base lengths of 3 inches and the prism has a height of 6 inches.

The company wants to use the package that has the least amount of wasted space.

Therefore, The company should choose the cylinder because it has approximately 11.6 in.3 less wasted space than the prism.

To learn more about the cylinder or a square prism visit:

https://brainly.com/question/4049983

#SPJ5

x2 + y2 =400 has center coordinates ____ and a radius equal to ___

Answers

Answer:

Answers are below

Step-by-step explanation:

The center coordinates of the circle are at (0,0).

The radius of the circle is 20

I graphed the circle on the graph below to show you how I got my answers.

The center of the equation x² + y² = 400 is, (0, 0).

And, The radius is 20.

What is Circle?

The circle is a closed two dimensional figure , in which the set of all points is equidistance from the center.

We have to given that;

The equation is,

⇒ x² + y² = 400

Now, We know that;

The standard for of the equation of circle is,

⇒ (x - h)² + (y - k)² = r²

Where, (h, k) is the center of the circle and 'r' is the radius of the circle.

Hence, By comparing we get;

⇒ x² + y² = 400

⇒ (x - 0)² + (y - 0)² = 20²

The center of the equation x² + y² = 400 is, (0, 0).

And, The radius is 20.

Learn more about the circle visit:

https://brainly.com/question/24810873

#SPJ2

The temperature fell from 0 Degrees Fahrenheit to 15 and one-half Degrees Fahrenheit below 0 in 5 and three-fourths hours. Wen tried to find the change in temperature per hour. Her work is shown below. Negative 15 and one-half divided by 5 and three-fourths = negative StartFraction 31 over 2 EndFraction divided by StartFraction 23 over 4 EndFraction = negative StartFraction 31 over 2 EndFraction times StartFraction 23 over 4 EndFraction = Negative StartFraction 713 over 8 EndFraction

Answers

Answer:

The correct answer will be:

[tex]-\dfrac{62}{23}[/tex]

Step-by-step explanation:

It is given that :

Initial temperature, [tex]T_1 = 0^\circ F[/tex]

Final temperature,

[tex]T_2 = -15\dfrac{1}{2}^\circ F\\\Rightarrow T_2 = -\dfrac{15\times 2+1}{2} ^\circ F\\\Rightarrow T_2 = -\dfrac{31}{2} ^\circ F[/tex]

Time taken :

[tex]5\dfrac{3}{4}\ hrs = \dfrac{5 \times 4+3}{4}\ hrs = \dfrac{23}{4}\ hrs[/tex]

Change in temperature per hour:

[tex]\dfrac{\text{Difference of temperature}}{\text{Total Time Taken}}\\\Rightarrow \dfrac{T_2-T_1}{\text{Total Time Taken}}[/tex]

Putting the values of temperatures and time:

[tex]\dfrac{\dfrac{-31}{2}-0}{\dfrac{23}{4}}\\\Rightarrow \dfrac{\dfrac{-31}{2}}{\dfrac{23}{4}}\\\Rightarrow \dfrac{-31 \times 4}{2 \times 23}} \text{---- Error done by Wen at this step}\\\Rightarrow \dfrac{-31 \times 2}{23}}\\\Rightarrow \dfrac{-62}{23}}[/tex]

The error done by Wen was during calculating the values of fraction.

So, the correct answer is :[tex]\frac{-62}{23}}[/tex] instead of [tex]\frac{-713}{8}[/tex]

Answer:

C. Wen did not take the reciprocal of the divisor

Step-by-step explanation:

Work out the value of x

Answers

Answer:

67.5

Step-by-step explanation:

right angle =90 degree

total = 360 degree

360-90=270

270÷4=67.5

What’s the correct answer for this?

Answers

Answer: choice B

Step-by-step explanation:

Events A and B are independent if the equation P(B)=P(B|A) or P(A∩B) = P(A) · P(B) holds true.

in this example

p(A)=1/6    {5}

p(B)=1/2   {1,3,5}

P(B|A)=1

so

P(B)≠P(B|A)

=>A and B are dependent

The sugar content of the syrup in canned peaches is normally distributed. Suppose that the variance is thought to be σ2=18 (milligrams)2. A random sample of n=10 cans yields a sample standard deviation of s=4.8 milligrams. Part 1 (a) Test the hypothesis H0:σ2=18 versus H1:σ2≠18 using α=0.05 Find χ02 .

Answers

Answer:

[tex]\chi^2 =\frac{10-1}{18} 23.04 =11.52[/tex]

The degrees of freedom are:

[tex] df =n-1=10-1=9[/tex]

Now we can calculate the critical value taking in count the alternative hypotheis we have two values:

[tex]\chi^2_{\alpha/2}= 2.70[/tex]

[tex]\chi^2_{1-\alpha/2}= 19.02[/tex]

Since the calculated value is between the two critical values we FAIL to reject the null hypothesis and we can't conclude that the true variance is different from 18

Step-by-step explanation:

Information given

[tex]n=10[/tex] represent the sample size

[tex]\alpha=0.05[/tex] represent the confidence level  

[tex]s^2 =4.8^2= 23.04 [/tex] represent the sample variance obtained

[tex]\sigma^2_0 =18[/tex] represent the value to verify

System of hypothesis

We want to verify if the true variance is different from 18, so the system of hypothesis would be:

Null Hypothesis: [tex]\sigma^2 = 18[/tex]

Alternative hypothesis: [tex]\sigma^2 \neq 18[/tex]

The statistic would be given by:

[tex]\chi^2 =\frac{n-1}{\sigma^2_0} s^2[/tex]

And replacing we got:

[tex]\chi^2 =\frac{10-1}{18} 23.04 =11.52[/tex]

The degrees of freedom are:

[tex] df =n-1=10-1=9[/tex]

Now we can calculate the critical value taking in count the alternative hypotheis we have two values:

[tex]\chi^2_{\alpha/2}= 2.70[/tex]

[tex]\chi^2_{1-\alpha/2}= 19.02[/tex]

Since the calculated value is between the two critical values we FAIL to reject the null hypothesis and we can't conclude that the true variance is different from 18

Based upon extensive data from a national high school educational testing​ program, the standard deviation of national test scores for mathematics was found to be 121 points. If a sample of 256 students are given the​ test, what would be the standard error of the​ mean?

Answers

Answer:

The standard error of the mean is 7.56

Step-by-step explanation:

The standard error of the mean is defined as the ratio of standard deviation to the square root of the sample size of a data. The standard error of the mean is mathematically expressed as shown below;

[tex]X = \frac{\sigma}{\sqrt{n} }[/tex]

X is the standard error of the mean

[tex]\sigma[/tex] is the standard deviation

n is the sample size.

Given;

[tex]\sigma = 121\\n = 256[/tex]

On substitution into the formula we have;

[tex]X = \frac{121}{\sqrt{256} }[/tex]

[tex]X = \frac{121}{16} }\\[/tex]

X ≈ 7.56

Find the time required for an investment of 5000 dollars to grow to 6100 dollars at an interest rate of 7.5 percent per year, compounded quarterly.

Answers

Answer:

The time (t) = 2.6 years

Step-by-step explanation:

To calculate the time for earning a compound interest, compounded on a certain amount of present value (PV), compounded periodically, the following formula is used:

[tex]FV=PV(1+\frac{i}{n} )^{n*t}[/tex]

where:

FV = future value = $6,100

PV = present value = $5,000

i = interest rate in decimal = 7.5% = 0.075

n = number of compounding periods per year = quarterly = 4 (4 quarters a year)

t = time of compounding in years = ???

Therefore the time is calculated thus:

[tex]6100=5000(1+\frac{0.075}{4} )^{4*t}[/tex]

[tex]6100=5000(1+0.01875 )^{4t}[/tex]

[tex]6100=5000(1.01875 )^{4t}[/tex]

Next, let us divide both sides of the equation by 5000

[tex]\frac{6100}{5000} = \frac{5000(1.01875)^{4t} }{5000}[/tex]

1.22 = [tex](1.01875)^{4t}[/tex]

Taking natural logarithm of both sides

㏑(1.22) = ㏑[tex](1.01875)^{4t}[/tex]

㏑(1.22) = 4t × ㏑(1.01875)

0.1989 = 4t × 0.01858

4t = [tex]\frac{0.1989}{0.01858} = 10.71[/tex]

∴ 4t = 10.71

t = 10.71 ÷ 4 = 2.6 years

The number of customers waiting for gift-wrap service at a department store is an rv X with possible values 0, 1, 2, 3, 4 and corresponding probabilities 0.1, 0.2, 0.3, 0.25, 0.15. A randomly selected customer will have 1, 2, or 3 packages for wrapping with probabilities 0.55, 0.35, and 0.1, respectively. Let Y = the total number of packages to be wrapped for the customers waiting in line (assume that the number of packages submitted by one customer is independent of the number submitted by any other customer).
(a) Determine P(X = 3, Y = 3), i.e., p(3,3). (Round your answer to four decimal places.)
(b) Determine p(4,11). (Round your answer to four decimal places.) p(4,11) = ?

Answers

Answer:

(a)p(3,3)=0.0416

(b)p(4,11)=0.0002

Step-by-step explanation:

Number of Customers, X

[tex]\left\begin{array}{|c|ccccc|}X&0&1&2&3&4\\P(X)&0.1&0.2&0.3&0.25&0.15.\end{array}\right[/tex]

Y = the total number of packages to be wrapped for the customers waiting in line

[tex]\left\begin{array}{|c|ccccc|}Y&1&2&3\\P(Y)&0.55&0.35&0.1\end{array}\right|[/tex]

a. P(X = 3, Y = 3)

p(3,3) means that there are 3 customers with one gift each

The probability of this event happening:

[tex]0.25 \times 0.55^3=0.0416[/tex]

p(3,3)=0.0416

b. p(4,11)

For 4 people to have a total package of 11, there must be 3 customers with 3 packages each and 1 customer with 2 packages,

The probability of this happening is:

[tex]p(4,11)=0.15\times^4C_1\times0.1^3\times0.35\\p(4,11)=0.0002[/tex]

A stem and leaf plot shows data in _____ order.

alphabetical
numerical
reverse

Answers

Numerical order !
this is for the extra letters lol

At a certain gas station 40% of the customers request regular gas, 35% request unleaded gas, and 25% request premium gas. Of those customers requesting regular gas, only 30% fill their tanks all the way up, while the remaining 70% only fill up part of their tank. Of those customers requesting unleaded gas, 60% fill their tanks all the way up, while of those requesting premium, 50% fill their tanks all the way up. If the next customer does not fill the tank all the way up (only fills it up part of the way), what is the probability that they requested regular gas?

Answers

Answer:

51.38% probability that they requested regular gas

Step-by-step explanation:

Bayes Theorem:

Two events, A and B.

[tex]P(B|A) = \frac{P(B)*P(A|B)}{P(A)}[/tex]

In which P(B|A) is the probability of B happening when A has happened and P(A|B) is the probability of A happening when B has happened.

In this question, we have that:

Event A: Not filling the tank

Event B: Regular gas

40% of the customers request regular gas

This means that [tex]P(B) = 0.4[/tex]

Of those customers requesting regular gas, 70% only fill up part of their tank.

This means that [tex]P(A|B) = 0.7[/tex]

Probability of not filling the tank:

70% of 40%(regular gas)

100 - 60 = 40% of 35%(unleaded gas).

100 - 50 = 50% of 25%(premium gas).

So

[tex]P(A) = 0.7*0.4 + 0.4*0.35 + 0.5*0.25 = 0.545[/tex]

What is the probability that they requested regular gas?

[tex]P(B|A) = \frac{0.4*0.7}{0.545}[/tex] = 0.5138

51.38% probability that they requested regular gas

IXL RELATED PLEASE HELP


In ΔDEF, the measure of ∠F=90°, DE = 82 feet, and EF = 55 feet. Find the measure of ∠E to the nearest tenth of a degree.

Answers

Answer:

To the nearest tenth of a degree

∠E= 47.9°

Step-by-step explanation:

ΔDEF , let's recognize that it's a right angle triangle because of the presence of ∠F=90°.

Other important measurements are

DE = 82 feet, and EF = 55 feet.

We are to find ∠E

Let's find the missing side first

Side DF

DF² = DE² - EF²

DF²= 82²_55²

DF²=6724-3025

DF²= 3699

DF= 60.82 feet

Let's use the sine formula to look for the angle.

DF/sin ∠E= DE/sin 90

Sin 90= 1

DE= 82

DF= 60.82

Sin ∠E= DF/DE

Sin ∠E= 60.82/82

Sin ∠E= 0.7417

∠E= sin^-1 (0.7417)

∠E= 47.8764

To the nearest tenth of a degree

= 47.9°

Solve 25 = 5x − 4.

one half
2
4
6

Answers

Answer:

29/5 =x

Step-by-step explanation:

25 = 5x − 4

Add 4 to each side

25+4 = 5x − 4+4

29 = 5x

Divide each side by 5

29/5 = 5x/5

29/5 =x

Answer:

The answer is 6

Step-by-step explanation:

:)

PLZZZ I need URGENT help with problems 1 and 2! I'm so confused. Thx!

Answers

Answer:Y=x3

2nd answer: Just add it all up and divide by how many things you had to add. For an example: (2+2+2+2)/4.

Step-by-step explanation:

the given equation has been solved on the table

Answers

Answer:

A. Step 2

Step by step explanation:

It was applied in Step 2 where 9 was added to both sides of the equation.

A circle has a circumference of 7,850. What is the radius of the circle?

Answers

Answer:

50

Step-by-step explanation:

7850/3.14

Square root 2500

50

7850/3.14 and then the square root 2500
50

This data set represents the number of

children in 8 families

4, 2, 1, 2, 4, 2, 6, 3

The mean of this data set is 3. What is the

Mean Absolute Deviation (MAD)?

A. 1.25

B. 8

C. 3.3

D. 3

Answers

Answer:

1.25

Step-by-step explanation:

Lets sum up the values

24

and then divide by their number (8)

we get 3 on average

now let's see how much each data point varies from that average (this is always a positive value)

1, 1, 2, 1, 1, 1, 3, 0

Let's sum and divide by 8 again

10/8 = 1.25

(would really, reallly appreciate the brainliest)

Other Questions
10. The measure of an interior angle of a regular polygon is 120. How many sides does the polygon have? A. 5 B. 6 C. 3 D 4o Someone help me please with those questions please HELP! ASAP! Jason practices his crossover dribble for x minutes per day on weekdays, and for three times as long on each weekend day. How many minutes did Jason practice in 4 weeks? 4. Graph the data in the table below. Which kind of function best models the data? Write an equation to model the data.012.34- 6- 912- 1518O quadratic; y = -x2 + 2x - 6linear; y = - 3x -6O quadratic; y = 32 - 2xexponential; y = -61.5% Work out the length of BC . 9. Given the following balanced chemicalequation,6NaOH + 3Cl2 - NaClO3 + 5NaCl + 3H20how many moles of NaCl can be formed if 6mol of NaOH were to react? Which central government agency was responsible to provide news on the war? choose the correct congruence statement for the figure shown What is the value of 7C2? A restaurant offers a $12 dinner special that has 5 choice for an appetizer, 13 choices for an entree and 4 choice for a dessert how many different meals are available when you select an appetizer an entree and dessert? yeah i need help with this Major developments during the civil rights movement? 18,500 g how many grams 22 g how many grams4.7 kg how many grams8.3 kg how many grams135,000mg how many gramsBEST ANSWER GETS BRAINLESST AND A FOLLOW YOU DON'T HAVE TO ANSWER ALL BUT AT LEAST DO SOME PLS ASP!(the pic shows you the same questions) Why does a predator kill its prey, when a parasite rarely kills its host? Anyone from the UK???? I have English exam tomorrow and I'm foreigener so English is it's not my native. I'll give u my mail or snaxp to contact easily. I hope someone can help. I'll make the brainliest answer the one who will help me tomorrow. Plus 50points aren't that bad yk :) PLEASE HELP! Use triangle ROE to find the following. Leave the answer in fraction form (no decimal answer) SHOW WORK Orion Iron Corp. tracks the number of units purchased and sold throughout each year but applies its inventory costing method at the end of the year, as if it uses a periodic inventory system. Assume its accounting records provided the following information at the end of the annual accounting period, December 31, 2012. Transactions Units Unit Cost a. Inventory, December 31, 2011 500 $ 10 For the year 2012: b. Purchase, April 11 800 8 c. Purchase, June 1 700 12 d. Sale, May 1 (sold for $38 per unit) 500 e. Sale, July 3 (sold for $38 per unit) 520 f. Operating expenses (excluding income tax expense), $19,000 Required: 1. Calculate the number and cost of goods available for sale. 2. Calculate the number of units in ending inventory. 3. Compute the cost of ending inventory and cost of goods sold under (a) FIFO, (b) LIFO, and (c) weighted average cost. (Do not round intermediate calculations. Round your final answers to the nearest dollar amount.) 4. Prepare an Income Statement that shows 2012 amounts under the FIFO method, LIFO method and weighted average method. How do the 9 dogs positions themselves around Napoleon at future meetings? A bag has 10 pens in it: 4 black, 3 blue, 2 green and 1 redYou randomly choose one pen from the bag and give it to Maria (who keeps it). You then randomly choose anotherpen from the bag and give it to Ahmed. What is the probability that Maria gets a black pen and Ahmed gets a redpen?45O0 1 / 2cationRiEw9Chip LMN PON. What is the value of x? Select one: a. 28 1/3 b. 36 c. 20 d. 25